Derivative of Integral: Is F'(x) = 2x sin(x^2) the Correct Answer?

Click For Summary
The derivative of the function F(x) = ∫^0_{x^2-1} (sin(t+1)/(t+1)) dt requires the application of the chain rule due to the variable limit of integration. The initial attempt at the derivative, F'(x) = -sin(x^2)/x^2, is incorrect because it overlooks the need to account for the derivative of the upper limit, which is a function of x. By applying the Fundamental Theorem of Calculus correctly, the derivative is found to be -sin(x^2)/x^2 multiplied by the derivative of the upper limit, resulting in -sin(x^2)/x^2 * 2x. This final expression can be simplified further, confirming the importance of careful application of calculus principles.
theRukus
Messages
49
Reaction score
0

Homework Statement


Find the derivative of the function
F(x) = \int^0_{x^2-1}\frac{sin(t+1)}{t+1}dt


Homework Equations





The Attempt at a Solution


F'(x) = -\frac{sin(x^2)}{x^2}

I'm just learning this and unsure if this is correct. It seems too easy?
 
Physics news on Phys.org
theRukus said:

Homework Statement


Find the derivative of the function
F(x) = \int^0_{x^2-1}\frac{sin(t+1)}{t+1}dt


Homework Equations





The Attempt at a Solution


F'(x) = -\frac{sin(x^2)}{x^2}

I'm just learning this and unsure if this is correct. It seems too easy?

Right. It's not as easy as you are making it. You need to use the chain rule.

F(x) = \int^0_{x^2-1}\frac{sin(t+1)}{t+1}dt = -\int_0^{x^2-1}\frac{sin(t+1)}{t+1}dt

The Fundamental Theorem of Calculus says that, if
F(x) = \int_0^x f(t)dt
then F'(x) = f(x)

Notice however, that one of your integration limits is not x, but is instead a function of x.

\frac{d}{dx}\int_0^{u} f(t)dt = \frac{d}{du}\int_0^u f(t)dt \cdot \frac{du}{dx}

Now the integral matches the form in the FTC.
 
So the answer would be,

-\frac{sin(x^2)}{x^2} \cdot 2x

Is this now correct?
 
\cdot for center dot.
 
theRukus said:
So the answer would be,

-\frac{sin(x^2)}{x^2} \cdot 2x

Is this now correct?
Looks good, but can be simplified a bit.
 
Question: A clock's minute hand has length 4 and its hour hand has length 3. What is the distance between the tips at the moment when it is increasing most rapidly?(Putnam Exam Question) Answer: Making assumption that both the hands moves at constant angular velocities, the answer is ## \sqrt{7} .## But don't you think this assumption is somewhat doubtful and wrong?

Similar threads

  • · Replies 6 ·
Replies
6
Views
2K
  • · Replies 23 ·
Replies
23
Views
2K
  • · Replies 105 ·
4
Replies
105
Views
6K
  • · Replies 15 ·
Replies
15
Views
2K
Replies
3
Views
2K
  • · Replies 11 ·
Replies
11
Views
3K
  • · Replies 3 ·
Replies
3
Views
2K
  • · Replies 5 ·
Replies
5
Views
2K
Replies
2
Views
1K
  • · Replies 2 ·
Replies
2
Views
1K